Analytic Integration of Function Containing the Exponential of an Exponential

Click For Summary
The discussion centers on the analytical integration of a complex function involving the exponential of an exponential. The integral in question is defined over the entire real line and incorporates parameters a, b, and L, which are positive constants. An attempt to change variables to u = e^{-θ} introduces complications, including a pole at x = 0 due to the Jacobian. Participants are seeking alternative methods for solving the integral without encountering these issues. The conversation highlights the challenges of integrating such intricate functions analytically.
junt
Messages
15
Reaction score
1

Homework Statement


Can this function be integrated analytically?

##f=\exp \left(-\frac{e^{-2 \theta } \left(a \left(b^2 \left(e^{2 \theta }-1\right)^2 L^2+16\right)-32
\sqrt{a} e^{\theta }+16 e^{2 \theta }\right)}{b L^4}\right),##
where ##a##, ##b## and ##L## are some real positive constants.

Homework Equations


This is the integral I am looking at:
##I=\int_{-\infty}^{\infty}\exp \left(-\frac{e^{-2 \theta } \left(a \left(b^2 \left(e^{2 \theta }-1\right)^2 L^2+16\right)-32
\sqrt{a} e^{\theta }+16 e^{2 \theta }\right)}{b L^4}\right) d\theta##

The Attempt at a Solution



One can change the coordinates ##u## to ##e^{-\theta}##, but then Jacobian will be inverse in ##x##, as result introduced a pole at ##x=0##. Does anyone know a better solution to it?
 
Physics news on Phys.org
junt said:

Homework Statement


Can this function be integrated analytically?

##f=\exp \left(-\frac{e^{-2 \theta } \left(a \left(b^2 \left(e^{2 \theta }-1\right)^2 L^2+16\right)-32
\sqrt{a} e^{\theta }+16 e^{2 \theta }\right)}{b L^4}\right),##
where ##a##, ##b## and ##L## are some real positive constants.

Homework Equations


This is the integral I am looking at:
##I=\int_{-\infty}^{\infty}\exp \left(-\frac{e^{-2 \theta } \left(a \left(b^2 \left(e^{2 \theta }-1\right)^2 L^2+16\right)-32
\sqrt{a} e^{\theta }+16 e^{2 \theta }\right)}{b L^4}\right) d\theta##

The Attempt at a Solution


One can change the coordinates ##u## to ##e^{-\theta}##, but then Jacobian will be inverse in ##x##, as result introduced a pole at ##x=0##. Does anyone know a better solution to it?
That's very difficult to read. I used \displaystyle in a couple of places each.

##\displaystyle f=\exp \left(-\frac{\displaystyle e^{-2 \theta } \left(a \left(b^2 \left(e^{2 \theta }-1\right)^2 L^2+16\right)-32
\sqrt{a} e^{\theta }+16 e^{2 \theta }\right)}{b L^4}\right) ##

##\displaystyle I=\int_{-\infty}^{\infty}\exp \left(-\frac{\displaystyle e^{-2 \theta } \left(a \left(b^2 \left(e^{2 \theta }-1\right)^2 L^2+16\right)-32
\sqrt{a} e^{\theta }+16 e^{2 \theta }\right)}{b L^4}\right) d\theta##
 
Question: A clock's minute hand has length 4 and its hour hand has length 3. What is the distance between the tips at the moment when it is increasing most rapidly?(Putnam Exam Question) Answer: Making assumption that both the hands moves at constant angular velocities, the answer is ## \sqrt{7} .## But don't you think this assumption is somewhat doubtful and wrong?

Similar threads

  • · Replies 5 ·
Replies
5
Views
2K
  • · Replies 4 ·
Replies
4
Views
1K
  • · Replies 11 ·
Replies
11
Views
2K
  • · Replies 22 ·
Replies
22
Views
3K
  • · Replies 5 ·
Replies
5
Views
2K
  • · Replies 5 ·
Replies
5
Views
2K
Replies
8
Views
2K
  • · Replies 3 ·
Replies
3
Views
2K
  • · Replies 47 ·
2
Replies
47
Views
4K
Replies
20
Views
2K